Sunteți pe pagina 1din 5

Solutions to Problems on Incidence Matrix

Problem 1. 8 singers participate in an art festival where m songs are performed. Each song is performed by 4 singers, and each pair of singers performs together in the same number of songs. Determine the minimum value of m. Solution. Notice that for every song performed, there are 4 = 6 ways to pair up the 2 four singers. Thus there were 6 pairs who performed together in each song. Suppose every pair of singer sang together in k songs. There are a total of 8 = 28 pairs of 2 singers, so by counting the total number of times each pair of singers perform, we have the equality 28k = 6m which means 14 | m thus m 14. Finally, we give a construction for m = 14. By numbering the singers from 1 to 8, divide them into 14 sets of 4 people each, to represent the singers for each of the 14 songs as follows: {1,2,3,4}, {3,4,5,6}, {5,6,7,8}, {1,2,7,8}, {1,2,5,6}, {2,3,6,7}, {3,4,7,8}, {1,4,5,8}, {1,3,5,7}, {2,4,6,8}, {1,3,6,8}, {2,3,5,8}, {2,4,5,7}, {1,4,6,7}. Problem 2. (Baltic Way 2001) A set of 8 problems was prepared for an examination. Each student was given 3 of them. No two students received more than one common problem. What is the largest possible number of students? Solution. Denote the problems by A, B,C, D, E, F, G, H, then 8 possible problem sets are ABC, ADE, AFG, BDG, BFH,CDH,CEF, EGH. Hence, there could be 8 students. Suppose that some problem (e.g., A) was given to 4 students. Then each of these 4 students should receive 2 different supplementary problems, and there should be at least 9 problems, a contradiction. Therefore each problem was given to at most 3 students, and there were at most 8 3 = 24 awardings of problems. As each student was awarded 3 problems, there were at most 8 students. Problem 3. 17 students took part in a mathematical contest with 9 problems. It is given that every problem was answered correctly by exactly 11 students. Prove that there were two students who, between them, solved all 9 problems. Solution. We set up an incidence matrix with columns P1 , P2 , . . . , P9 to represent the problems and rows a1 , a2 , . . . , a17 to represent the students. The cell in row i and column j is lled with 1 if student ai solved problem Pj , and 0 otherwise. Since every problem was answered correctly by exactly 11 students, there must be exactly six 0s in each column. The number of column pairs of 0s is therefore 6 9 = 135. Now 2 suppose that no two students solved all 9 problems between them. This means that for every pair of students ai and a j , 1 i < j 17, there is at least one problem they both didnt solve, contributing to at least one column pair of 0s. Thus total number of column pairs 17 = 136, a contradiction. 2 Problem 4. (SMO(S)2001 Round 2) The numbers {1, 2, . . . , n2 } are randomly arranged on a n2 n2 board, such that each number appears exactly n2 times. Prove that there is a row or a column that contains at least n distinct numbers. Solution. Set up an incidence matrix with columns corresponding to the numbers 1 to n2 and rows r1 , r2 , . . . , rn2 , c1 , c2 , . . . , cn2 to represent the rows and columns of the board
Prepared by Ho Jun Wei Hwa Chong Math Olympiad Programme (Open)

respectively. Fill cell in column k with a 1 if the number k appears on that particular row ri or column c j of the board, and 0 if it does not appear in that row/column of the board. Every number appears exactly n2 times on the board. If a certain number appears in exactly a columns and b rows of the board, then we must have ab n2 a + b 2n by AM-GM inequality. Then in every column of our incidence matrix, there must be at least 2n 1s, thus the total number of 1s in the matrix 2n3 . Since there are 2n2 rows in our incidence matrix, by pigeonhole principle, there must be at least one row that contains at least n 1s. This means that at least one row or column on the board contains n distinct numbers. Problem 5. In a chess club, n people gathered to play chess against each other, as they pleased. No two people played against each other more than once. At the end of the day, it was observed that a total of 3n games had been played. Moreover, if we chose any two players, say A and B, there would be at most one other player who had played with both A and B. Prove that n > 30. Solution. Set up an incidence matrix with n rows and n columns, and ll the cell in row i and column j with a 1 if person i played with person j, and 0 otherwise (a person cannot play against himself). Let ci represent the number of 1s in column i. Each match contributes to two 1s, thus ci = 6n. By Jensens inequality, the function x x2 x 2 = 2 is concave upwards, thus the total number of column pairs is
n i=1

ci 2

ci n

=n

6 = 15n. 2

Also, for every pair of people A and B, at most one person played with both A and B, thus between every two rows, there can be at most one column pair, so total number of column pairs n = n(n1) . Combining both inequalities, we obtain 15n n(n1) 2 2 2 which implies n 31 as desired. Problem 6. (SMO(O)2004 Round 2) Let m, n be integers so that m n > 1. Let F1 , . . . , Fk be a collection of n-element subsets of {1, ..., m} so that Fi Fj contains at most 1 element, 1 i < j k. Show that k m(m 1) n(n 1)

Solution. Let S be the set of all possible two-element subsets {X,Y } {1, 2, 3, . . . , m}. Clearly |S| = m . Each Fi contains n elements, there are exactly n elements in S that 2 2 are subsets of Fi . Also, note that |Fi Fj | 1, thus no element in S can be a subset of two sets Fi and Fj . This means k n |S| k m(m1) . 2 n(n1) Problem 7. (SMO(O)2006 Round 2) Let n be a positive integer. Let S1 , S2 , . . . , Sk be a collection of 2n-element subsets of {1, 2, 3, 4, . . . , 4n 1, 4n} so that Si S j contains at most n elements for all 1 i < j k. Show that k 6(n+1)/2 Solution. Let A be the set of all possible (n + 1)-element subsets of {1, 2, 3, 4, . . . , 4n 4n 2n 1, 4n}. Clearly |A| = n+1 . Since each Si contains 2n elements, there will be n+1 elements in A that are subsets of Si . Furthermore, each element in A can be a subset of

Prepared by Ho Jun Wei Hwa Chong Math Olympiad Programme (Open)

at most one Si , since Si S j < n + 1. Thus we obtain k 2n 4n |A| = k n+1 n+1


4n n+1 2n n+1

4n (4n 1) . . . 3n . 2n (2n 1) . . . n

Finally, we prove by induction that 4n (4n 1) . . . 3n 6(n+1)/2 . 2n (2n 1) . . . n For n = 1 the result is trivially true. Now suppose it holds for n = r. 4r (4r 1) . . . 3r 6(r+1)/2 , 2r (2r 1) . . . r (4r + 4) (4r + 3) . . . (3r + 3) (4r + 4)(4r + 3)(4r + 2)(4r + 1)(r) = (2r + 2) (2r + 1) . . . (r + 1) (3r + 2)(3r + 1)(3r)(2r + 2)(2r + 1) (4r + 4)(4r + 3)(4r + 2)(4r + 1)(r) (r+1)/2 6 (3r + 2)(3r + 1)(3r)(2r + 2)(2r + 1) (4r + 4)(4r + 2) (4r + 3)(4r + 1) r (r+1)/2 = 6 (2r + 2)(2r + 1) (3r + 2)(3r + 1) 3r 16 1 4 6(r+1)/2 9 3 64 (r+1)/2 = 6 27 6 6(r+1)/2 Let = = 6(r+2)/2 The induction is complete. Thus k 4n (4n 1) . . . 3n 6(n+1)/2 for all integers n. 2n (2n 1) . . . n

Problem 8. Consider the set A = {1, 2, 3, . . . , 100}. Let S1 , S2 , S3 , . . . , S5n be subsets of A, such that every element of A appears in exactly 4n of these subsets. Prove that there exists i, j, k, such that Si S j Sk = A. Solution. We set up an incidence matrix with 100 columns and rows S1 , S2 , . . . , S5n . The cell in row Si and column j is lled with 1 if j appears in Si and 0 otherwise. Since each number appears in exactly 4n subsets, thus every column has exactly 4n 1s and n 0s. We dene a column triplet as a triplet of 0s occuring in a column. Thus if there are ci 0s in column i, the number of column triplets in that column is ci . The 3 total number of column triplets is therefore 100 n . Suppose to the contrary that for 3 all i < j < k, we have Si S j Sk = A, then each selection of (i, j, k) must admit at least one column triplet. Thus there are at least 5n column triplets. We therefore obtain the 3 inequality 5n n 100 3 3 but this is a contradiction, since
5n 3 n 3

100

5n(5n 1)(5n 2) 100, n(n 1)(n 2)

5n(5n 1)(5n 2) 5n(5n 5)(5n 10) = 125. n(n 1)(n 2) n(n 1)(n 2) 3

Prepared by Ho Jun Wei Hwa Chong Math Olympiad Programme (Open)

Problem 9. (USAMO 1984) A math exam has two papers, where each paper consists of at least one question and both papers have 28 questions altogether. Each pupil attempted 7 questions. Each pair of questions was attempted by just two pupils. Show that one pupil attempted either zero or at least four questions in the rst paper. Solution. Each pupil attempts 7 questions and hence 21 pairs of questions. There are 28 = 378 pairs of questions in total and each is attempted by 2 pupils. So there 2 must be 3782 = 36 pupils. Suppose n pupils solved question 1. Each solved 6 pairs 21 involving question 1, so there must be 3n pairs involving question 1. But there are 27 pairs involving question 1, so n = 9. The same applies to any other question. So each question was solved by 9 pupils. Suppose the result is false. Suppose there are m questions in the rst paper, and that the number of pupils solving 1, 2, 3 questions in the rst paper is a, b, c respectively. So a + b + c = 36, a + 2b + 3c = 9m. Now consider pairs of problems in the rst paper. There are m(m1) such pairs. Pupils solving just 1 solve no pairs, those solving 2 solve 1 2 pair and those solving 3 solve 3 pairs, so we have b + 3c = m(m 1). Solving for b we get 29 23 b = 2m2 + 29m 108 = 2(m )2 < 0. 4 8 This is a contradiction, so the result must be true. Problem 10. (China 1994) Given that S = {1, 2, 3, . . . , 10} and A1 , A2 , . . . , Ak are subsets of S satisfying |Ai | = 5 for 1 i k and |Ai A j | 2 for 1 i < j k, nd the maximum value of k. Solution. We construct an incidence matrix of 10 columns and rows A1 to Ak . We ll each cell in column i and row A j with 1 if the number i appears in A j , and 0 otherwise. Let ci be the number of 1s in column i. Since |Ai | = 5, we have a total of 5k 1s in the matrix, thus ci = 5k. Next we count column pairs of 1s. For each pair (Ai , A j ), we have at most 2 column pairs since they contain at most two common element. Therefore, 10 ci k 2 2 2 = k(k 1). i=1 However, by Jensens inequality, the function total number of column pairs is
10 i=1 x 2

x2 x 2

is concave upwards, thus the

ci 2

10

ci 10

= 10

k 2

k k = 5( )( 1). 2 2

Combining the inequalities, we obtain k k 5( )( 1) k(k 1) k 6, 2 2 Finally, we can easily verify that the following 6 sets satisfy the condition. {1, 2, 3, 4, 5}, {1, 2, 6, 7, 8}, {1, 3, 6, 9, 10}, {2, 4, 7, 9, 10}, {3, 5, 7, 8, 10}, {4, 5, 6, 8, 9} Thus the maximum value of k is 6.

Prepared by Ho Jun Wei Hwa Chong Math Olympiad Programme (Open)

Problem 11. (IMO 2001 Problem 3) Twenty-one girls and twenty-one boys took part in a mathematical competition. It turned out that each contestant solved at most six problems, and for each pair of a girl and a boy, there was at least one problem that was solved by both the girl and the boy. Show that there is a problem that was solved by at least three girls and at least three boys. Solution. We set up a 2121 matrix with rows corresponding to the boys b1 , b2 , . . . , b21 and columns corresponding to the girls g1 , g2 , . . . , g21 . For each pair (bi , g j ), we ll the corresponding cell with question number of a problem that both bi and g j solved (for instance, if b1 and g1 both solved question 9, we ll this cell with a 9). Next, consider the row b1 . Since each boy solved at most six problems, there are at most six different question numbers that appear in the row b1 . For every question number that appears more than three times in that row, we shade the cell blue. By pigeonhole principle, there will be a problem that was solved by at least 4 girls. Thus there are at most 5 question numbers in the row whose cells are unshaded, and each of these question numbers occupies at most 2 cells (otherwise it will be shaded), thus there will at least be 11 cells shaded blue in the row b1 . This argument extends to all rows, thus there will be at least 21 11 = 232 blue cells. Next, considering the column g1 , we shade a cell pink if the question number appears at least three times in that column. By a similar argument, there are at least 11 pink cells in column g1 , and at least 21 11 = 232 pink cells in the entire matrix. Since there are only 21 21 = 441 < 232 + 232 cells in the matrix, there must be a cell that is shaded both pink and blue. This problem was solved by at least 3 girls (since it is shaded blue) and at least 3 boys (since it is shaded pink).

Prepared by Ho Jun Wei Hwa Chong Math Olympiad Programme (Open)

S-ar putea să vă placă și